A grocery shopper tosses a 6 kg bag of rice into a stationary 27 kg grocery cart. The bag hits the cart with a horizontal speed of 8.0 m/s toward the front of the cart. What is the final speed of the cart and the bag? Round to the hundredths place

Answers

Answer 1

Given,

The mass of the bag, m=6 kg

The mass of the cart, M=27 kg

The initial velocity of the cart, u=0 m/s

The initial velocity of the bag, v=8.0 m/s

From the law of conservation of momentum, the sum momentum of the bag and the momentum of the cart before the bag hits the cart must be equal to the momentum of the bag and the cart after the bag hits the cart.

Thus,

[tex]mv+Mu=(M+m)V[/tex]

Where V is the velocity of the bag and the cart after the bag hits the cart.

On rearranging,

[tex]V=\frac{mv+Mu}{(M+m)}[/tex]

On substituting the known values,

[tex]\begin{gathered} V=\frac{6\times8.0+0}{(27+6)} \\ =\frac{48}{33} \\ =1.45\text{ m/s} \end{gathered}[/tex]

Thus the final speed of the cart and the bag is 1.45 m/s


Related Questions

Do you know the answer for these? [ showing the work] I'm trying to figure out the answers to the bullet points under your mission section.

Answers

Answer:

Explanation:

• Orbital Radius,:

We are already told that the altitude of the satellite is 300,000 meters. Having this information in hand, it is easy to find the radius of orbit using the following relation:

[tex]R_{\text{satellite}}=A+R_{\text{earth}}[/tex]

where A is the altitude.

Now we know that A = 300,000 m and earth radius = 6.37 * 10^6 m; therefore, the orbital radius of the satellite is

[tex]R_{\text{satellite}}=3\cdot10^6+6.37\cdot10^6[/tex]

[tex]\boxed{R_{\text{satellite}}=9.37\times10^6m}[/tex]

which is our answer!

• Velocity:

We are told that the velocity of the satellite is given by

[tex]v=\sqrt[]{G\frac{m_E}{R_{\text{satellite}}}}[/tex]

where G is the gravitational constant and m_E is the mass of the earth.

Substituting the numerical values for these constants gives

[tex]v=\sqrt[]{(6.67\times10^{-11})\frac{5.98\times10^{24}}{9.37\times10^6}}[/tex]

Using a calculator we evaluate the above to be:

[tex]\boxed{v=6.52\cdot10^3m/s}[/tex]

which is around 6.5 km per second!

• Orbital Period:

The orbital period T of the satellite is given by

[tex]T=2\pi\sqrt[]{\frac{R^3_{satellite}}{Gm_E}}[/tex]

putting in the numerical values for the constants gives

[tex]T=2\pi\sqrt[]{\frac{(9.37\times10^6)^3}{(6.67\times10^{-11})(5.98\times10^{24})}}[/tex][tex]\boxed{T=9024s\approx2.5hr}[/tex]

Hence, the period of satellites orbit is only 2.5 hours! This means that we can see the same satellite multiple times in the night sky if it is observable!

• Orbital Path:

The problem with satellites is that since they are travelling so fast, they don't get to observe one location on earth for a long time. One solution to this is to place the satellites into something called the geosynchronous orbit. In such an orbit, the period of the satellite matches the earth's period of rotation. This way, when observed from the earth, the satellite looks stationary, but in fact, it is travelling with the earth in the same orbital period. Such a satellite can be launched to observe locations along the arctic and the antarctic circles to obtain substantial data.

A cart of mass m is attached to a perfect spring and oscillates on a frictionless surface, as shown. The amplitude of the oscillation is given by x. The kinetic energy of the cart has a maximum value at(a) position A(b) position B(c) position C(d) positions A and C

Answers

Kinetic energy is the energy an object possess during motion. Therefore, the kinetic energy of the cart reaches it maximum kinetic energy as the cart passes through the equilibrium position. Therefore, the position is at C.

42) When you weigh yourself on good old terra firma (solid ground), your weight is 142 lb. In an elevator your apparent weight is 121 lb. What are the direction and magnitude of the elevator’s acceleration?

Answers

We are given the following information

Actual weight = 142 lb.

Apparent weight = 121 lb.

Direction of the elevator's acceleration:

Notice that the apparent weight (121 lb.) is less than the actual weight (142 lb.)

This means that the direction of the elevator's acceleration must be downward since a downward acceleration

results in an apparent weight less than the actual weight.

Magnitude of the elevator’s acceleration:

There are two forces acting on you in the elevator.

The applied force of scale acts upward and the force due to gravity acts downward.

Let us apply Newton's 2nd law of motion to find the magnitude of the elevator’s acceleration.

[tex]\begin{gathered} \sum F_y=m\cdot a \\ W_a-W=m\cdot a \\ a=\frac{W_a-W}{m} \\ a=\frac{W_a-W}{\frac{W}{g}} \\ a=\frac{W_a-W}{W}\cdot g \\ a=\frac{121-142}{142}\cdot9.81 \\ a=-1.5\; \; \frac{m}{s^2} \end{gathered}[/tex]

Notice that the negative sign represents a downward acceleration as we discussed earlier.

When we are interested in the magnitude, we only consider the numerical value, not the direction.

Therefore, the magnitude of the elevator’s acceleration is 1.5 m/s²

According to Bernoulli's principle, what type of relationship exists between a fluid's velocity and its pressure?O an inverse relationshipO a direct relationshipO a balanced relationshipO an unbalanced relationship

Answers

To find:

The relation between the pressure and the velocity of the fluid.

Explanation:

The Bernoulli's principle states that the increase in the velocity of the fluid occurs simultaneously with the decrease in the static pressure of the fluid.

That is, the velocity of the fluid is inversely proportioinal to the pressure of the fluid.

Final answer:

Thus the relationship between a fluid's velocity and its pressure is an inverse relationship.

Therefore, the correct answer is option A.

A 1-cm high object is placed 10 cm in front of a concave mirror with a focal length 15 cm. Determine the image distance and the magnification. Enumerate the characteristics of the image produced. Must follow GRESA format.Given:Required:Equation(s) Needed:Solution:(a)(b)(c)Final answer:L-ocationO-rientation:S-izeT-ype:

Answers

Given:

object size: 1 cm

object distance (do): 10 cm

mirror focal length (f): 15 cm

Required:

Image distance (di),

magnification

image characteristics.

Equations needed:

[tex]\begin{gathered} \frac{1}{d_i}+\frac{1}{d_o}=\frac{1}{f} \\ \text{magnification}=-\frac{d_i}{d_o} \end{gathered}[/tex]

Solution:

[tex]\begin{gathered} \frac{1}{d_i}+\frac{1}{10}=\frac{1}{15} \\ \frac{1}{d_i}=\frac{1}{15}-\frac{1}{10} \\ \frac{1}{d_i}=\frac{2}{30}-\frac{3}{30} \\ \frac{1}{d_i}=\frac{-1}{30}_{} \\ d_i=-30\text{ cm} \\ \\ \text{magnification}=-\frac{-30}{10}=3 \end{gathered}[/tex]

Location: 30 cm "inside" the mirror (di = -30)

Orientation: Upright (magnification > 0)

Size: 3 cm (object size multiplied by magnification)

Type: virtual image (di < 0)

A car with a mass of 1200kg is driving in circular path with radius of 65m at a constant speed of 5.5 m/s. What is the magnitude of the net force on the car? a) 102N b) 14182 N c) 6600 N d) 78000 N e) 558 N

Answers

Take into account that due to the speed of the car is constant, the net force on the car is the force due to the centrifugal force, which is given by the following formula:

[tex]F=ma_c=m\cdot\frac{v^2}{r}[/tex]

where:

m: mass of the car = 1200 kg

v: speed = 5.5 m/s

r: radius of the circular trajectory = 65 m

replace the previous values of the parameters into the formula for F:

[tex]F=1200kg\cdot\frac{(5.5m/s)^2}{65m}=558.46N\approx558N[/tex]

Lightning is useful for human beings.Give reason.

Answers

ANSWER and EXPLANATION

Lightning is an occurrence where electric charges of very high voltage are discharged from a cloud and unto the ground or another cloud.

One of the surprising benefits of lightning is that it helps in the breakdown of Nitrogen in the atmosphere. Nitrogen makes up about 70% of the atmosphere but it is in a form that is unusable to plants in the ground. Lightning strikes help to breakdown the NItrogen in the atmosphere, serving as natural fertilizer to plants.

The other benefit of lightning is that it helps in the production of ozone, which is a vital gas that helps protect us from dangerous ultraviolet rays from the sun.

What defines momentum and what causes it to change?

Answers

Momentum can be said to be the quantity of motion that a moving body possess.

Momentum is a vector quantity, which means it has both magnitude and direction.

Momentum is the product of the mass of the object and the velocity.

It is expressed as:

p = mv

Where:

p is the momentum

m is the mass object.

v is the velocity of the object

Momentum is changed due to the force that acts on the moving body for a given amount of time, this causes a change in velocity which in turn causes the momentum to change.

Therefore, if the there is a change in the velocity of an object, it will cause the momentum to change.

ANSWER:

• Momentum is the product of the mass and velocity of a moving body.

• A change in the velocity due to the force that acts on the object will change the object's momentum.

A softball hit by a bat accelerates at 2729 m/s2. If the net force acting on the softball is 508 N, what is the softball's mass?
Round your answer to the nearest 0.001. Do not add units.

Answers

0.186 g is the softball's mass when A softball hit by a bat accelerates at 2729 m/s2.

net force= 508 N,

acceleration=2729 m/s2

softball's mass?

Force=ma

m = force/acceleration

m= 508/2729

m=0.186 g

Mass is one of the fundamental quantities in physics and the most fundamental property of matter. The quantity of matter in a body is referred to as its mass. The SI unit of mass is the kilogram.

The mass of a body is constant at all times. only when a body receives or loses a considerable amount of energy, which happens seldom. For instance, a nuclear reaction causes the mass of the material to decrease because a very little amount of matter is converted into a very high amount of energy.

To know more about  mass visit : https://brainly.com/question/15067814

#SPJ9

What is the wavelength range of electromagnetic radiation thehuman eye can detect?a 4 x 10-7 m to 8 x 10-7 mb 9 x 10-8 m to 11 x 10-9 mC 1x 10-4 m to 3 x 10-5 md 6 x 10-5 m to 7 x 10-5 m

Answers

The wavelength range of electromagnetic radiation the human eye can detect is:

a 4 x 10-7 m to 8 x 10-7 m

Give reasons. #) Egg sinks in freshwater but floats in saturated solution of salt and water

Answers

Given

Egg sinks in freshwater but floats in saturated solution of salt and water​

To find

Give reason

Explanation

We know the density of egg is more than the density of fresh water while it is less than the density of saturated solution

Thus the upward force ( upthrust) from the freshwater couldnot balance the downward for the edd and it sinks.

In case of saturated solution, the egg's downward force is less than the upward force of the saturated solution and thus it floats

A car speeds up uniformly while rounding a turn. The car's angular speed increases from 0.54 radians per second to 0.96 radians per second as it turns through 1.40 radians. What is the car's angular acceleration? Include units in your answer. Answer must be in 3 significant digits.

Answers

Given data:

* The initial angular speed of the car is,

[tex]\omega_i=0.54\text{ rad/s}[/tex]

* The final angular speed of the car is,

[tex]\omega_f=0.96\text{ rad/s}[/tex]

* The angular displacement of the car is,

[tex]\theta=1.4\text{ radians}[/tex]

Solution:

By the kinematics equation, the angular acceleration of the car in terms of the angular displacement is,

[tex]\omega^2_{\text{f}}-\omega^2_i=2\alpha\theta[/tex]

where,

[tex]\alpha\text{ is the angular acceleration,}[/tex]

Substituting the known values,

[tex]\begin{gathered} 0.96^2-0.54^2=2\times\alpha\times1.4 \\ \alpha=\frac{0.96^2-0.54^2}{2\times1.4} \\ \alpha=\frac{0.9216-0.2916}{2.8} \\ \alpha=\frac{0.63}{2.8} \end{gathered}[/tex]

By simplifying,

[tex]\alpha=0.225rads^{-2}[/tex]

Thus, the angular acceleration of the car is 0.225 radians per second squared.

A car traveling at 11.6 meters per second crashes into a barrier and stops in 0.287 meters. What force must be exerted on a child of mass 21.2 kilograms to stop him or her in the same time as the car? Include units in your answer. Answer must be in 3 significant digits. Hint: This is an impulse-momentum theorem problem.

Answers

[tex]\begin{gathered} \text{For car} \\ v_1=11.6\text{ m/s} \\ \Delta x=0.287m \\ v_2=\text{ 0 m/s} \\ t=\text{?} \\ To\text{ find t} \\ v^2_2=v^2_1+2a\Delta x \\ \text{Solving a} \\ v^2_2-v^2_1=2a\Delta x \\ a=\frac{v^2_2-v^2_1}{2\Delta x} \\ a=\frac{(0m/s)^2-(11.6m/s)^2}{2(0.287m)} \\ a=\frac{0m^2/s^2-134.56m^2/s^2}{0.574m} \\ a=\frac{-134.56m^2/s^2}{0.574m} \\ a=-234.42m/s^2 \\ \text{Then} \\ t=\frac{v_2-v_1}{a} \\ t=\frac{0\text{ m/s-11.6m/s}}{-234.42m/s^2} \\ t=\frac{\text{-11.6m/s}}{-234.42m/s^2} \\ t=0.0495\text{ s} \\ \text{For child} \\ m=21.2\text{ kg} \\ v_1=11.6\text{ m/s} \\ v_2=\text{ 0 m/s} \\ t=0.0495\text{ s} \\ F=\text{?} \\ F=\frac{P_2-P_1}{t} \\ P_2-P_1=mv_2-mv_1=m(v_2-v_1),\text{ then} \\ F=\frac{m(v_2-v_1)}{t} \\ F=\frac{(21.2kg)(0\text{ m/s-11.6m/s})}{0.0495\text{ s}} \\ F=\frac{(21.2kg)(-11.6\text{ m/s})}{0.0495\text{ s}} \\ F=\frac{-245.92\operatorname{kg}\text{ m/s}}{0.0495\text{ s}} \\ F=-4968\text{ N} \\ \text{The force to stop the child is 4968 N. The negative means } \\ \text{the force is opposite to the movement} \end{gathered}[/tex]

Which of the following phenomena provide evidence for the wave theory of light?a) Wavelength theoryb) Color Theoryc) The photoelectric effectd) Diffraction

Answers

The double slip experiement shows that light can display properties of particles and waves. The wave part of this experiment is due to the phenomenon of diffraction that appears in the behaviour of light whe it passes through a slit. This means that light has wave propeties. Therefore, the right asnwer is d) diffraction.

The ejection of electrons from metals illuminated with different frequencies of light provides evidence for the particle theory of light.

What is the photoelectric effect?

The effect is often defined as the ejection of electrons from a metal plate when light falls on it.

In a broader definition, the radiant energy may be infrared, visible, or ultraviolet light, X-rays, or gamma rays; the material may be a solid, liquid, or gas; and the released particles may be ions (electrically charged atoms or molecules) as well as electrons. The phenomenon was fundamentally significant in the development of modern physics because of the puzzling questions it raised about the nature of light—particle versus wavelike behavior—that was finally resolved by Albert Einstein in 1905. The effect remains important for research in areas from materials science to astrophysics, as well as forming the basis for a variety of useful devices.

Discovery: The photoelectric effect was discovered in 1887 by the German physicist Heinrich Rudolf Hertz. In connection with work on radio waves, Hertz observed that, when ultraviolet light shines on two metal electrodes with a voltage applied across them, the light changes the voltage at which sparking takes place.

Applications: Devices based on the photoelectric effect have several desirable properties, including producing a current that is directly proportional to light intensity and a very fast response time. One basic device is the photoelectric cell or photodiode. Originally, this was a phototube, a vacuum tube containing a cathode made of metal with a small work function so that electrons would be easily emitted

To learn more about the photoelectric effect visit,

https://brainly.com/question/22054853

I have some problems applying the formulas to solve physics problems. I understand all the concepts needed, but just freeze when I see questions, especially when it comes to trying to combine linear and rotational conceptsA uniform, 255 N rod that is 1.90 m long carries a 225 N weight at its right end and an unknown weight W toward the left end (Figure 1). When W is placed 60.0 cm from the left end of the rod, the system just balances horizontally when the fulcrum is located 75.0 cm from the right end.1) Find W.2) If W is now moved 30.0 cm to the right, how far must the fulcrum be moved to restore balance?

Answers

ANSWER

[tex]\begin{gathered} 1)\text{ }214.90\text{ }N \\ \\ 2)\text{ }0.09\text{ }m \end{gathered}[/tex]

EXPLANATION

First, let us make a sketch of the diagram showing the distances on the rod:

1) Since the fulcrum is balanced, the center of gravity of the system will be at the fulcrum.

The center of gravity (in the horizontal is given by:

[tex]x=\frac{W_1x_1+W_2x_2+W_3x_3}{W_1+W_2+W_3}[/tex]

where W1 = the weight on the right end = 225 N

W2 = the weight of the rod = 255 N

W3 = the weight place on the left = W

x1 = the position of W1 (taking the left as the origin) = 1.90 m

x2 = the position of the center of mass of the rod = x1/2 = 0.95 m

x3 = the position of W from the left end = 0.60 m

x = position of center of gravity of the rod from the left end i.e. at the fulcrum = 1.90 - 0.75 = 1.15 m

Now, substitute the values given in the question and solve for W:

[tex]\begin{gathered} 1.15=\frac{(225*1.90)+(255*0.95)+(W*0.60)}{225+255+W} \\ \\ 1.15=\frac{427.5+242.25+0.60W}{480+W} \\ \\ 1.15(480+W)=669.75+0.60W \\ \\ 552+1.15W=669.75+0.60W \\ \\ 1.15W-0.60W=669.75-552 \\ \\ 0.55W=117.75 \\ \\ W=\frac{117.75}{0.55} \\ \\ W=214.09\text{ }N \end{gathered}[/tex]

That is the value of W.

2) Now, W is moved 30.0 cm (0.30 m) to the right.

This implies that:

[tex]x_3=0.60+0.30=0.90\text{ }m[/tex]

Since the other values (including W) do not change, we can now solve for x, which is the new center of gravity:

[tex]\begin{gathered} x=\frac{(225\times1.90)+(255\times0.95)+(214.09\times0.90)}{225+255+214.09} \\ \\ x=\frac{427.5+242.25+192.681}{694.09}=\frac{862.431}{694.09} \\ \\ x=1.24\text{ }m \end{gathered}[/tex]

Therefore, the fulcrum must be moved:

[tex]\begin{gathered} 1.24\text{ }m-1.15\text{ }m \\ \\ 0.09\text{ }m \end{gathered}[/tex]

The fulcrum should be moved 0.09 m to the right (since the W is moved to the right).

Discuss how a very large electric field is generated by using an induction coil

Answers

The working of induction coils involves principles of electromagnetic induction in which magnetic field is used to produce voltage. The magnetic field is produced by an alternating current which passes through the coil. \The induction coil has primary and secondary coils. By the principle of mutual induction, electric current is induced in the secondary coil if the magnetic flux due to the primary coil changes or it is interrupted. Small current is sent into the primary coil which creates the magnetic field. A sudden interruption of the voltage going into the primary coil causes the magnetic field to collapse abruptly. This abrupt collapse of the magnetic field generates high voltage pulses in the secondary coil by the process known as magnetic induction. The secondary coil has more turns of wire. Thus, large electric fields are produced.

A vector has an x-componentof -309 m and a y-componentof 187 m.Find the magnitude and direction of thevector.

Answers

In order to find the magnitude we will follow the next formula

[tex]M=\sqrt[]{C^2_x+C^2_y}[/tex]

Cx= -309

Cy=187

We substitute the values

[tex]M=\sqrt[]{(-309)^2+(187)^2}=361.18m[/tex]

Then for the direction we need o use the next formula

[tex]\theta=\tan ^{-1}(\frac{Cy}{Cx})[/tex]

We substitute the variables

[tex]\theta=\tan ^{-1}(\frac{187}{-309})=-31.181[/tex]

In order to obtain the direction we need to add to the angle 180°

the direction is -31.181+180=148.82°

ANSWER

magnitud =361.18

direction =148.82°

Stephan has to drive 8 miles south and 3 miles east to get to work.
What is his displacement?
What is his distance?​

Answers

The distance and displacement of Stephan are 11 miles and 8.54 miles respectively.

Distance is a measurement of how far apart two things or points are, either numerically or occasionally qualitatively.

To get to work, Stephan must travel 3 miles east and 8 miles south.

Therefore, the total distance traveled by Stephan will be:

d = 3 miles + 8 miles

d = 11 miles

The displacement is the shortest distance between the starting point and the endpoint.

If Stephan first travels South and then east, the turn will make a 90-degree angle between the two distances.

Therefore, by using the Pythagoras theorem the displacement is:

D² = 3² + 8²

D² = 9 + 64

D² = 73

D = √73

D = 8.54 miles

Stephan's displacement and distance are 11 miles and 8.54 miles respectively.

Learn more about displacement here:

brainly.com/question/14422259

#SPJ9

When you look at an image of yourself in a plane mirror, the image is:A.real.B.enlarged.C.reduced.D.virtual.

Answers

Since the light does not pass through the object the image will be virtual . [Option D]

Lighting seeks the most direct path to the ground. True O False

Answers

Lighting is basically the discharge of electricity that does not care where it strikes. It basically seeks the shortest or the most direct path to the ground.

Therefore, the given statement is a true statement.

Answer:

TRUE

Explanation:

Negatively charged particles in the out most energy levels of the electron cloud ?

Answers

Electrons are negatively charged particles in the out most energy levels of the electron cloud

An atom is made up of 3 particles , which are electron , proton and neutron . Where electron is a negatively charged particle and revolves around the nucleus in definite orbits whereas protons and neutrons are positively charged particles and neutral respectively ., which resides inside the nucleus

Electrons are negatively charged subatomic particles found in the outermost regions of atoms .

To learn more about electrons here:

https://brainly.com/question/1255220

#SPJ1

A/An _____ is described as a type of circuit in which there is only a single current path.parallel circuitseries circuitshort circuitvoltage divider

Answers

ANSWER

series circuit

EXPLANATION

A series circuit is a type of circuit where all the components are connected one after the other. For example:

So there are no alternative paths for the current. Hence we say that there's a single current path.

A ballistic pendulum consists of a 1.25-kg block of wood that is hanging from the ceiling in such a way that when a bullet enters it, the block’s change in height can be recorded as it swings. A bullet having a mass of 6.25-grams and unknown velocity strikes the block and becomes imbedded in it. The impulse imparted to the block causes it to swing in such a way that its height increases by 7.15 cm.1. What was the change in potential energy of the block/bullet combo after the collision?2. What was the speed of the block/bullet combo immediately after the collision (and before it beganto swing)?3. What was the speed of the bullet before entering the block of wood?

Answers

1.

The potential energy is defined as:

[tex]U=mgh[/tex]

The change in potential energy is:

[tex]\Delta U=U_f-U_i[/tex]

the initial height of the block/bullet is zero, then we have that:

[tex]\begin{gathered} \Delta U=(1.25+0.00625)(9.8)(0.0715)-0 \\ \Delta U=0.8802 \end{gathered}[/tex]

Therefore, the change in potential energy is 0.8802 J

2.

We know that the energy is conserved, this means that the kinetic energy inmediately after collision has to be equal to the potential energy at the 7.15 cm heigh, then we have:

[tex]\begin{gathered} \frac{1}{2}mv^2=0.8802 \\ \frac{1}{2}(1.25625)v^2=0.8802 \\ v=\sqrt[]{\frac{2\cdot0.8802}{1.25625}} \\ v=1.184 \end{gathered}[/tex]

Therefore, the velocity at this moment is 1.184 m/s

3.

From conservation of momentum we know that:

[tex]m_bv_b=mv[/tex]

then:

[tex]\begin{gathered} (0.00625)v_b=(1.25625)(1.184) \\ v_b=\frac{(1.25625)(1.184)}{0.00625} \\ v_b=237.98 \end{gathered}[/tex]

Therefore the speed of the bullet is 237.98 m/s

what is the theory E=MC^2?

Answers

E = M C ^2 is Albert Einstein's theory of special relativity.

It states that the mass multiplied by the speed of light squared equal to the kinetic energy of that body.

m= mass

C = speed of light

Radioactive radium has a half-life of approximately 1599 years. What percent of a given amount remains after 100 years? (Round your answer to two decimal places.)

Answers

[tex]P(t)=1*0.5^{\frac{t}{h}}[/tex]

Where:

P(t): percentage of initial remaining

t: time elapsed

h: half life of substance

Given the variables we know, we can solve for P(t).

[tex]P(t)=0.5^{\frac{100}{1599}}[/tex]

Plugging this into a calculator, we get that

P(t) = 0.95757733 = 95.757733% remaining

Dispersion occurs forGroup of answer choicesmonochromatic light in reflectionmonochromatic light in refractionpolychromatic light in reflectionpolychromatic light in refraction

Answers

Given:

Dispersion

Required: To choose the correct option.

Explanation:

Reflection is the process when a light ray strikes a surface and returns back in the same medium.

Refraction is the process of bending light when light travels from one medium to another.

Monochromatic light is light that has only one color.

Polychromatic light is the light that constitutes multiple colors in the same source.

When dispersion occurs light travels from one medium to another, and it bends after entering the second medium.

As polychromatic has multiple colors and each color corresponds to a particular wavelength.

The bending of light also differs by wavelength, so each color will bend at different angles.

Final Answer: The dispersion occurs for polychromatic light in refraction.

A bumper car with a mass of 150 kg is moving forward at 10m/s. It collides with a stationary bumper car with a mass of 209 kg. After the collision, the 200 kg bumper car moves forward at 8.0 m/s. Does the 150 bumper car continue to move forward or backward after the collision?

Answers

Given,

Mass of the bumper car is 150 kg.

It moves with a speed 10 m//s.

The mass of the stationary bumper car is 200 kg.

After collision,

The 200 kg bumper moves it a velocity of 8.0 m/s.

By conservation of mass,

[tex]\begin{gathered} 150\times10=200\times8+150v \\ \Rightarrow v=-0.66\text{ m/s} \end{gathered}[/tex]

Thus the 150 kg bumper car moves backward after collision.

A model rocket is launched with an initial upward velocity of 215 ft/s. The rockets height h in feet) after t seconds is given by the following.h=215t-16t^2Find all values for t for which the rockets height is 97 feet.Round your answer(s) to the nearest hundredth.

Answers

Given,

The initial velocity of the rocket, u=215 ft/s

The equation which gives the height of the rocket at any given instant of time,

[tex]h=215t-16t^2[/tex]

The height of the rocket, h=97 ft.

The given equation is a quadratic equation. On rearranging the above equation and substituting the value of h,

[tex]\begin{gathered} 16t^2-215t+h=0 \\ 16t^2-t+97=0 \end{gathered}[/tex]

A quadratic equation is written as,

[tex]ax^2+bx+c=0[/tex]

Comparing the two equations,

x=t, a=16, b=-215, c=97.

The solution of a quadratic equation is given by,

[tex]x=\frac{-b\pm\sqrt[]{b^2-4ac}}{2a}[/tex]

On substituting the known values in the above equation,

[tex]\begin{gathered} t=\frac{-(-215)\pm\sqrt[]{(-215)^2-4\times16\times97}}{2\times16} \\ =\frac{215\pm\sqrt[]{40017}}{32} \\ =12.97\text{ or}\pm0.47\text{ } \end{gathered}[/tex]

Thus, the values of t are 12.97 s or 0.47 s.

That is, the rocket will be at a height of 97 feet at t=12.97 s or t=0.47 s

Alex goes cruising on his dirt bike. He rides 700 m north, 300 m east, 400 m north, 600 m west, 1200 m south, 300 m east, and finally 100 m north. What is his total displacement?

Answers

ANSWER

[tex]0\operatorname{km}[/tex]

EXPLANATION

First, let us make a sketch of the question:

From the diagram:

=> black line represents North

=> green line represents East

=> blue line represents West

=> red line represents South

From the diagram, we see that at the end of his journey, he returns to his start point.

Since displacement is the measure of the change in the position of an object and the boy's position did not change after the journey, his displacement is:

[tex]0\operatorname{km}[/tex]

Where does electricity come from I.n Your own words

Answers

Electricity is electrons moving through a conductor.

Voltage pushes electrons through the conductor to create an electric current.

Electricity comes from Power stations , that have generators that produce electricity.

Generators use different sources of energy such as water flow, fossil fuel, etc.

Other Questions
I just need to know the answer for question 12 Solve the puzzleABCDEFGHIJKLMNOP1101134NUCLEAR _ A ____L6 19 10 42 26 3 249 4320 2613 3E4 13 53LUN AR L.4 1F Athineon 2020ALL-4 4 123 20 4 19 6 15 3 2920 7 20 26NE1320 66BQR20 2E9 13 16ARRE24 3 2 2 9 13 7N(__.10 22 20 20 16 9R24 20 2 13STUVWXYZ191E10 20 24 25 20 16 13 7LE10 20 19 25 4 13 1665AREN25 3 2 13 6 15 22 2010 22 9L EN.4 7 2 13 6led maths &simalein subjectof teacherderCholeping inor roleel andsponson help to raisecontrol, Privateof local authorityAcademies-Taken outthat are runreligious ethosT X Por Han spent a total of $221.76, before tax, on bags of chips for the basketball team, and each bag cost $3.52. What is the total number of bags of chips that Han bought? - 16mn-(-25mn)+(-7mn)Which of the following is equivalent to the expression above? What were the primary causes of the French Revolution? select four options.1.lack of representation for the lower classes.2.the establishment of a state religion. 3.oppressive leadership.4.high taxes on the lower class.5.high rates of poverty. The product of the ages (in days) of two newborn babies Simran and Jessie in two dayswill be 48 more than the product of their ages today. How old are the babies if Jessie is 2days older than Simran? The probability that VSU and KSU both win a basketball game in the same week is 47%. Theprobability that VSU wins is 50%. What is the probability that KSU will win given that VSUhas already won? Simple calculator test - determine the angle, , to the nearest whole degree from each of the following:A. cos() = 0.2079B. tan() = 0.2126C. sin() = 0.5446 Consider the quadratic function.What are the x-intercepts and y-intercept?What is the equation of the axis of symmetry?What are the coordinates of the vertex?Graph the function on the coordinate plane. Include the axis of symmetry. What are the advantages and disadvantages of public transit?Give me one advantage or disadvantage of public transit and use the template below to give me 2-4 sentences?Supporting Paragraph #1 Topic Sentence (The main point of your first support paragraph):____________________________________________________________________________ Which values has three significant figures?14001001101.22310 An equilibrium mixture, at 529C in a 1208-mL container, involving the chemical systemN2(g) + 3H2(g) == 2NH3(g)is found to contain 3.62 g of N2, 0.773 g of H2, and 0.498 g of NH3. Calculate the equilibrium constant (Keq expressed in terms of the molar concentrations) at the given temperature. the sum of 2 numbers is 75. the second number is 3 less than twice the first. find the numbers what is your stance on the subject physical education being undervalued what is the value of x would make lines l and m parrallel How would you prepare 250 mL of 0.125 M HCl from concentrated HCl (aq) that is 38.0% by mass with a density of 1.19 g/mLnote: find moles in 250 mL of 0.125 HCl whats the variance and the standard deviation of the data set 6,9,2,5,4,10,3,12,2,7,7,8 What are the coordinates of point P on the directed line segment from R to Q such that 5 Pis 6 the length of the line segment from R to Q? Round to the nearest tenth, if necessary. A (-3.5, 2.3 ) B. (3.5, -2.3 ) C. (-3, 2) D. (3, 2) The ratio of the length of an airplane wing to its width is 9 to 1. If the length of a wing is 43.9 metrs, how wide must it be? The airplane wing must be ____ meters wide. (Round to the nearest hundredth.) Consider the equation 2x+5y=1Identify the slope and y intercept